solve the equation x + 5 = 12

Answers

Answer 1

Answer:

x = 7

Step-by-step explanation:

x + 5 = 12

Subtract 5 from both sides

5 - 5 cancels out

12 - 5 = 7

We're left with x = 7


Related Questions

Rewrite as a power of 10: 1/10

Answers

Answer:

Step-by-step explanation:

You have to know how negative exponents "work" to understand this concept.

[tex]\frac{1}{10}=10^{-1[/tex] because if you want to make a negative exponent positive you put what the exponent is on under a 1. It follows then that you can go backwards from that and rewrite positive fractions with negative exponents.

Is (18,-4) a solution to the equation y = -6x - -87? yes no​

Answers

Replace x with 18, solve the equation. If it equals -4 it’s a solution.

Y = -6(18) - -87

Y = -108 + 87

Y = -21

-21 does not equal -4 so (18,-4) is not a solution.

please help i have class soon :(

John used digits from 0 to 6 only to create a three-digit PIN code. He forgot his PIN code. He remembers that his code contains different ascending digits. What is the maximum number of combinations John has to check to find his PIN code?

Answers

this was incorrect.

im sorry :(

Answer:

35

Step-by-step explanation:

We know that the numbers have to be in ascending order. If, for example, the first digit is 0, and second digit is 1, then we would have 5 other numbers in the ones place that would be larger. If the second digit is 2, then would would have 4 other numbers that would be larger, and so on. So if 0 is the first digit, then we would have 5+4+3+2+1=15 combinations. If 1 is the first digit, then we would have 4+3+2+1 combinations. Eventually, we would get 35.

f(x)=2x+3/4x+5 find f(-9)

pls solve it ​

Answers

Answer:

15/31

Step-by-step explanation:

substitute -9 were there's x in the function

f(-9)=2(-9)+3/4(-9)+5

=-18+3/-36+5

=-15/-31

the negatives cancel giving you

15/31

Two square based pyramids are joined, total volume is 2700mm, perpendicular height of top pyramid is 16mm, perpendicular height of bottom pyramid is 20mm, length and width of joint base area both x, find x. Please help me

Answers

Answer: 15 m

Step-by-step explanation:

Given

Total volume of the combined pyramid is [tex]V=2700\ mm^3[/tex]

Height of top and bottom pyramid is

[tex]h_t=16\ mm[/tex]

[tex]h_b=20\ mm[/tex]

If the base has a side length of x, its area must be [tex]x^2[/tex]

Volume of square prism is given by

[tex]\Rightarrow V=\dfrac{1}{3}Bh\quad [\text{B=base area}]\\\\\text{Total volume will be the sum of the two pyramids}\\\\\Rightarrow 2700=\dfrac{1}{3}\times x^2\times 16+\dfrac{1}{3}\times x^2\times 20\\\\\Rightarrow 2700=\dfrac{1}{3}\times x^2\times (16+20)\\\\\Rightarrow x^2=225\\\Rightarrow x=15\ mm[/tex]

Thus, the value of [tex]x[/tex] is 15 m.

Which of the following is ordered pair for point C?

Answers

Answer:

B. (4,2)

Step-by-step explanation:

Answer:

B(4,2)

Step-by-step explanation:

as you can see that if want to find coordinates u should know that the position of x and y is (x,y). So u can that c on the x is lower than 5 so that u can say it is 4 and y is too far away from 5 also u it will be 2.

through: (4,- 4), perp. to y = -4x - 2

Answers

Answer:

Step-by-step explanation:

y = -4x - 2

slope of line = -4

slope of perpendicular to line = ¼

Point-slope equation for line of slope ¼, passing through (4,-4):

y+4 = ¼(x-4)

In slope-intercept form:

y = ¼x - 5

Please help it’s needed

Answers

very easy    

[tex]\displaystyle\ (f-g)(x)=f(x)-g(x)=4x^2-5x-3x^2-6x+4=\boxed{x^2-11x+4}[/tex]

[tex]\\ \sf\longmapsto f(x)=4x^2-5x[/tex]

[tex]\\ \sf\longmapsto g(x)=3x^2+6x-4[/tex]

Now

[tex]\\ \sf\longmapsto (f-g)(x)[/tex]

[tex]\\ \sf\longmapsto f(x)-g(x)[/tex]

[tex]\\ \sf\longmapsto 4x^2-5x-(3x^2+6x-4)[/tex]

[tex]\\ \sf\longmapsto 4x^2-5x-3x^2-6x+4[/tex]

[tex]\\ \sf\longmapsto x^2-11x+4[/tex]

Does the point (0, 37) satisfy the equation y = 79x - -37? yes no​

Answers

Answer:

yes

Step-by-step explanation:

y = 79x - -37

37 = 79(0) - - 37

37 = 0 - - 37

37 = 37

Quadrilateral A B C D is shown. The uppercase right angle, angle A, is 79 degrees.
What are the remaining angle measures if the figure is to be a parallelogram?

m∠B =
°

m∠C =
°

m∠D =
°

Answers

Answer:

m∠B =  

✔ 101

°

m∠C =  

✔ 79

°

m∠D =  

✔ 101

°

Step-by-step explanation:

Answer:

The answer above is right!

The correct answers are:

First box: option C. 101

Second box: option B. 79

Third box: option C. 101

Step-by-step explanation:

Just got it right on edge - Hope it helps :)

Brainliest would be greatly appreciated :D

Expand the following using the Binomial Theorem and Pascal’s triangle (2x − 3y)^4

Answers

Answer:

Step-by-step explanation:

Binomial Expansion:

[tex](2x-3y)^4 \\\\= 4C_0(2x)^4 (-3y)^0 + 4C_1(2x)^3 (-3y)^1 + 4C_2(2x)^2(-3y)^2 \\\\+ 4C_3(2x)^1 (-3y)^3 + 4C_4(2x)^0 (-3y)^4\\\\=16x^4 + 4(8x^3)(-3y) + 6(4x^2)(9y^2) + 4(2x)(-27y^3) + 81y^4\\\\=16x^4 - 96x^3y + 216x^2y^2-216xy^3 +81y^4[/tex]

Pascal's Triangle:

  Zero row                        1

 First row                  1                 1

 Second row          1           2             1

 Third  row         1         3              3          1

 Fourth row    1       4            6           4          1

What is the value of x in the equation 0.7x – 1.4 = –3.5?

1. -7
2. -3
3. 7
4. 3

Answers

Answer:

the answer is -3

Step-by-step explanation:

0.7x - 1.4 = -3.5

add 1.4 to both sides of the equation

you're left with 0.7x = -2.1

then divide both sides by 0.7

you're left with your answer of -3

WILL GRANT BRAILIEST PLZ REAL ANSWERS ONLY OR REPORTING!​

Answers

Hello,

If the question is simplify then

we suppose x not equal to 5 and x not equal to -5

[tex]\dfrac{x^2-10x+25}{(x-5)(x+5)} \\\\=\dfrac{(x-5)^2}{(x-5)(x+5)} \\\\=\dfrac{x-5}{x+5} \\\\[/tex]

else

if the question is to find the euclidian 's quotient then

[tex]\dfrac{x^2-10x+25}{(x-5)(x+5)} \\\\= 1 + \dfrac{-10x+30}{x^2-25} \\\\=1-\frac{10}{x+5} \\[/tex]

euclian's quotient is 1

remainder is -10/(x+5)

Drag each condition to the correct location on the table.
Match each condition to the number of triangles that can be constructed to fit the condition.
condition A: one side
measuring 4 inches,
another side measuring
8 inches, and a third
side measuring 10 inches
condition B: one angle
measuring 60°, another
angle measuring 40°, and
a third angle measuring 90°
condition C: one side
measuring 4 inches,
another side measuring
8 inches, and the angle
between them measuring 70°
condition D: one side
measuring 5 inches,
another side measuring
7 inches, and a third
side measuring 12 inches
condition E: one angle
measuring 30°, another
angle measuring 80°,
and the length of the
included side measuring
8 inches
condition F: one angle
measuring 40°, another
angle measuring 80°, and
a third angle measuring 60° I don't know how to do a pic, but it is no triangle one triangle and many triangle

Answers

Answer:

Condition A= 320 inches

Condition B= 190 Degrees F

Condition C= 2240 inches

Condition D= 350 inches

Condition E= 110 Degrees F 8 inches

Condition F= 180 Degrees F

Step-by-step explanation:

4 inches

8 inches

70 inches

60 D

40 D

90 D

4 inches

8 inches

70 D

5 inches

7 inches

12 inches

30 D

80 D

8 inches

40 D

80 D

60 D

Answer:

Condition A= 320 inches

Condition B= 190 Degrees F

Condition C= 2240 inches

Condition D= 350 inches

Condition E= 110 Degrees F 8 inches

Condition F= 180 Degrees F

Step-by-step explanation:

probability, please help i will give brainliest

Answers

Answer:

Step-by-step explanation:

2 colours were taken

Graph: Y - 3 = 1/2 (x+2)

Answers

Answer:

see graph

Step-by-step explanation:

Personally I put into slope intercept form before graphing

y - 3 = 1/2 (x+2)

y -3 = 1/2x + 1

y = 1/2x + 4

4 is y-intercept    

1/2 is slope. [tex]\frac{Rise= 1}{Run= 2}[/tex]


The diagram shows a cylinder of diameter 6 cm and height 20 cm what is the volume in cm3

Answers

Answer:

565.2cm³

Step-by-step explanation:

the radius= 6/2= 3 cm

the height= 20cm

the volume= 3.14× 3²×20

= 3.14×180= 565.2 cm³

Check the picture below.

Can someone help me with this math homework please!

Answers

Answer:

1st option.

0 to 100 miles per hour

Step-by-step explanation:

as we can see the question states that a particular car' s gas mileage DEPENDS upon its speed.

Since, the independent variable is the domain of the function the speed of the car will act as the domain of the given function.

out of all the option option 1 gives us values of speed( cause its unit is miles/ hour - unit of speed).

so the domain is

0 to 100 miles per hour.

Answer:

(A) 0 to 100 miles per hour

Step-by-step explanation:

The domain is the independent variable, or the input. The speed is the independent variable because the gas mileage DEPENDS on it. That means the gas mileage is the dependent variable.

The only answer that is related to the speed is the first answer choice.

Hope that helps (●'◡'●)

10
What are the x- and y-coordinates of point E, which
partitions the directed line segment from A to B into a
ratio of 1:2?
B(-4,9)
9
8
X E
(
mn)(x2 – xı) + x
7-
6
5
4
mm. )(x2 - y) + y
3
-7 -6 -5 4 -3 -2 -14
2 3 4 5 6 7
0 (0, 1)
0 (-1,3)
O(-2,5)
(1,0)
X
-2.
3
A(2,-3)
19

Answers

Answer:

Step-by-step explanation:

The formulas to find the x and y coordinates of E are:

[tex]x=\frac{bx_1+ax_2}{a+b}[/tex]  and  [tex]y=\frac{by_1+ay_2}{a+b}[/tex]  where x1, x2, y1, and y2 are from the coordinates of A and B, and a = 1 (from the ratio) and b = 2 (from the ratio). Filling in to find x first:

[tex]x=\frac{2(2)+1(-4)}{1+2}=\frac{4-4}{3}=0[/tex]  and now for y:

[tex]y=\frac{2(-3)+1(9)}{1+2}=\frac{-6+9}{3}=\frac{3}{3}=1[/tex]

The coordinates of E are (0, 1).

Given:

The points are A(2,-3) and B(-4,9).

The point E divides the segment AB in 1:2.

To find:

The coordinates of point E.

Solution:

Section formula: If a point divides a line segment in m:n, then the coordinates of the point is:

[tex]Point=\left(\dfrac{mx_2+nx_1}{m+n},\dfrac{my_2+ny_1}{m+n}\right)[/tex]

Using the section formula, the coordinates of point E are:

[tex]E=\left(\dfrac{1(-4)+2(2)}{1+2},\dfrac{1(9)+2(-3)}{1+2}\right)[/tex]

[tex]E=\left(\dfrac{-4+4)}{3},\dfrac{9-6}{3}\right)[/tex]

[tex]E=\left(\dfrac{0)}{3},\dfrac{3}{3}\right)[/tex]

[tex]E=\left(0,1\right)[/tex]

Therefore, the coordinates of the point E are (0,1).

Please hurry I will mark you brainliest

Answers

Answer:

D. -1/7

Step-by-step explanation:

Substitute and solve

Answer:

[tex]-\frac{1}{7}[/tex]

Step-by-step explanation:

When given the following expression,

[tex]\frac{a+c}{a^2-c^2}[/tex]

With the information that the values (a = -2) and (c = 5), one is asked to evaluate the expression. One's first instinct is probably to substitute the values into the expression and solve, however, a faster approach is to simplify the expression. The denominator is the difference of squares, thus one can rewrite it as the product of two linear expressions. Then one can simplify it by canceling out like terms in the denominator and the numerator. Finally, one can then substitute the values of the (a) and (c) into the simplified expression and solve.

[tex]\frac{a+c}{a^2-c^2}[/tex]

[tex]=\frac{a+c}{(a+c)(a-c)}[/tex]

Cross out like terms in the numerator and denominator,

[tex]=\frac{a+c}{(a+c)(a-c)}[/tex]

[tex]=\frac{1}{a-c}[/tex]

Now substitute the values of (a) and (c) into the expression and simplify to evaluate,

[tex]=\frac{1}{a-c}[/tex]

[tex]=\frac{1}{(-2)-(5)}\\\\=\frac{1}{-2-5}\\\\=-\frac{1}{7}[/tex]

PLEASE HELP ME SOMEONE I NEEDDDDDDD HELP PLEASE QUICK!!!!!!!!

Answers

Answer:

2/60 = 1/30 = 3.3%

Step-by-step explanation:

Jesse spends 1/2 of his pocket money on Monday.
On Tuesday, he spends 2/3 of what is left.
On Wednesday, he spends 1/4 of what remains.
What fraction of the pocket money does he have left? Choose the most
reasonable answer

Answers

Answer:

The fraction of the pocket money she left is 1/8.

Step-by-step explanation:

Let the total pocket money is p.

Spent on Monday = p/2

Amount left = p - p/2 = p/2  

Spent on Tuesday = 2/3 of p/2 = p/3

Amount left = p/2 - p/3 = p/6

Spent on Wednesday = 1/4 of p/6 = p/24

Amount left = p/6 - p/24 = p/8

So, the fraction of the pocket money she left is 1/8.  

if equation by parabola is given by, p(x) = x^2 -5x + 6 then its factors are:
A. (x-3)
B. (x-2)
C. Both (a) and (b)
D. None of the above

Answers

Answer:

x^2-2x-3+6=0

(x-2x)-(3x+6)=0

x(x-2)-3(x-2)=0

(x-3)(x-2)=0

x-3=0 or x-2=0

x=3or x=2

Select the correct answer from each drop-down menu.
The equation of a line is 3/5x+1/3y=1/15
The x-intercept of the line is
and its y-intercept is


Answers

Answer:x intercept is where the line is crossing the x axis or where y=0

y intercept is where the line crosses the y axis or where x=0

to find them, set other ting to zero

3/5x+1/3y=1/15

xintercept, set y=0

3/5x+1/3(0)=1/15

3/5x+0=1/15

3/5x=1/15

times 5/3 both sides

x=1/9

xintercept is x=1/9

yintercet

set x=0

3/5(0)+1/3y=1/15

0+1/3y=1/15

1/3y=1/15

times 3/1 both sides

y=1/5

yintercept is y=1/5

xint is x=1/9

yint is y=1/5

Step-by-step explanation:

The equation of line is ( 3/5 )x+ ( 1/3)y = ( 1/15 ) , where x-intercept of the line is ( 1/9 , 0 ) and the y-intercept of the line is ( 0 , 1/5 )

What is an Equation of a line?

The equation of a line is expressed as y = mx + b where m is the slope and b is the y-intercept

And y - y₁ = m ( x - x₁ )

y = y-coordinate of second point

y₁ = y-coordinate of point one

m = slope

x = x-coordinate of second point

x₁ = x-coordinate of point one

The slope m = ( y₂ - y₁ ) / ( x₂ - x₁ )

Given data ,

Let the equation of line be represented as A

( 3/5 )x+ ( 1/3)y = ( 1/15 )

To find the x-intercept of the line, we set y = 0 and solve for x:

3/5x + 1/3(0) = 1/15

3/5x = 1/15

x = (1/15) ÷ (3/5) = 1/9

So the x-intercept of the line is (1/9, 0)

To find the y-intercept of the line, we set x = 0 and solve for y:

3/5(0) + 1/3y = 1/15

1/3y = 1/15

y = (1/15) ÷ (1/3) = 1/5

So the y-intercept of the line is (0, 1/5)

Hence , the equation of line is solved

To learn more about equation of line click :

https://brainly.com/question/14200719

#SPJ2

the median of 2,8,6,10,4,12​

Answers

Answer: 6

Step-by-step explanation: 6 is a median

Hey there!

To find your mean you have to put the numbers from descending (least/decreasing) to ascending (greater/increasing) order

Median is also understood as the “middle number”

2, 8, 6, 10, 4, 12​

= 2, 4, 6, 8, 10, 12

Based on that information, you have in your new data set you have 2 pairs in the middle (6 & 8), so you’ll have to total/average of the number. We have to DIVIDE them by 2 because it’s 2 numbers

6 ends the descending set and 8 starts the ascending set

Median = 6 + 8 / 2

6 + 8 = 14

= 14/2

= 7

Answer: therefore your MEDIAN is most likely 7

Good luck on your assignment and enjoy your day!

~Amphitrite1040:)

The number line shows the car's starting point and the ending point.
Find the distance between them.

Answers

Answer:

-12/7

Step-by-step explanation:

[tex] \frac{ - 5}{7} - \frac{7}{7} [/tex]

[tex] \frac{ - 5 - 7}{7} [/tex]

[tex] \frac{ - 12}{7} [/tex]

Third option is the answer.

Consider the equation: x^2 - 4x + 4 = 2x

Rewrite the equation by completing the square:
Your equation should look like (x+a)^2 = b or (x-c)^2 = d
______

What are the solutions to the equation? (1 right answer!)

Answers

Answer:

(x+3)^2=5

Step-by-step explanation:

x^2-4x+4=2x

x^2-6x+4=0

x^2-6x+9-5=0

(x-3)^2-5=0

(x-3)^2=5

The product of three different positive integers is equal to 7^3. What is the sum of the three integers?

Answers

Answer:

sum of three positive integer=57

Step-by-step explanation:

let x,y,z be three positive no.

given :-

xyz=7³

xyz=343

xyz=1×7×49

x+y+z=(1+7+49)

sum=57

Answer:

343

Step-by-step explanation:

7^3

change 7^3 to a natural number or enter 7^3 on a calculator

7×7×7

=343

factors of 343=1,7,49,343

1×7×49=343

1+7+49=57

Which of the following are integers?


100
78
35
5



There can be more than one

Answers

Answer: 100, 78, 35, 5 (all of them!)

Explanation: integers are any number what isn’t a fraction :)

Have a lovely day!
100 78 35 5 all of them

pls help:( algebra 1 btw


17(2 + 12g) + 14

Answers

Answer:

204g + 48

Step-by-step explanation:

simplifying the expression

Answer: 204g+48


Step 1: Rewrite equation

To start off, let’s rewrite the equation to better understand the problem we are working with.

17(2+12g)+14

Step 2: Distribute

In this type of equation, we will not find an exact answer. Rather, we will simplify the equation by distribution and combining like terms. Let’s start by distributing. To do so, multiply the numbers outside of the parentheses with the numbers on the inside of the parentheses. This will be 17•2 and 17•12g. Let’s do so now.

17(2+12g)+14
34+204g+14

Step 3: Combine like terms

As the final step, we need to combine like terms. This means we add/subtract terms that share the same variable, or lack of. In this case, we need to add 34 and 14 together. Let’s do this now.

34+204g+14
204g+48

This is your final answer. 204g+48 is the new equation. Hope this helps! Comment below for more questions.
Other Questions
In addition to highly repeated DNA sequences and unique (single-copy) DNA sequences, a third class of DNA exists. What is it called, and what types of elements are involved in it c) Solar energy is the source of all forms of energy.give reasons A regression was run to determine whether there is a relationship between hours of tv watched per day(x) and number of sit-ups a person can do (y). The results of the regression are given below. Use this to predict the number of sit-ups a person who watches 11 hours of tv can do Y=ax+b A=-1.341 B=32.234 R=-0.896 A car took 5 minutes to travel from town A to town B which are 12km apart.Calculate the average speed of the path. Write an expression that evaluates to True if the string associated with s1 is greater than the string associated with s2 If f(x) = 3 - 4x, find f(1+a)I am in the need of assistance thank you ! find the 9th and 15th terms of the following geometric sequence 2, -4, 8, -16 QuestionPrice of Book $14.99 9.99Copies Sold at That Price 100,000 174.000Total Revenue at That Price 1,499,000 1,738,000Based on the data in the table and using the midpoint formula, between a price of $9.99 and $14.99, the price elasticity of demand for books isA) -0.74.B) -1.16.C) -1.35.D) -14.8. Please helpPlease answer correctly Piaget's nursemaid told his family that she saved his life from an attempted kidnapping. Piaget believed the nursemaid and later recalled details of the event, even though it never actually took place. Piaget's ability to recall an event that never took place is demonstrative of: chemistry definition of Which of these men was a leading land developer in Denver?Charles GoodnightGeorge Armstrong CusterJohn Wesley IliffWilliam Larimer CORRECT ANSWERS ONLYYYAnna and her best friend David are both 20 years old with similar fitness lifestyles. Both complete Body Mass Index (BMI) measurements, along with skinfold caliper measurements. They are both found to have the same BMI, but Anna's skinfold caliper shows a slightly higher amount of total body fat. This is likely becauseA. she is femaleB. she is more athleticC. she is tallerD. she measured incorrectly A satellite is launched to orbit the Earth at an altitude of 2.90 x10^7 m for use in the Global Positioning System (GPS). Take the mass of the Earth to be 5.97 x 10^24 kg and its radius 6.38 x10^6 m.Required:What is the orbital period of this GPS satellite? 20 This diagram shows an enzyme-substrate complex.Structure XStructure ZStructure YmStep 1Step 2Step 3Which is represented by Structure X? Evaluate the expression shown below and write your answer as a fraction in simplest form. What ordered pairs are the solutions of the system of equations shown in the graph below? what is rule of law Help me plz to find product Find the area of the parallelogram